MS3Q

अब Quizwiz के साथ अपने होमवर्क और परीक्षाओं को एस करें!

The nurse is formulating a teaching plan according to evidence-based breast cancer screening guidelines for a 50-year-old woman with low risk factors. Which diagnostic methods should be included in the plan? (Select all that apply.) a. Annual mammogram b. Magnetic resonance imaging (MRI) c. Breast ultrasound d. Breast self-awareness e. Clinical breast examination

ANS: A, D, E Guidelines recommend a screening annual mammogram for women ages 40 years and older, breast self awareness, and a clinical breast examination. An MRI is recommended if there are known high risk factors. A breast ultrasound is used if there are problems discovered with the initial screening or dense breast tissue.

A 72-year-old woman is being assessed by the nurse for an annual physical. Which finding is of concern to the nurse? a. Thinning of pubic hair b. Increased size of the uterus c. Decreased size of the clitoris d. Loss of tone of the pelvic ligaments

ANS: B An increased size of the uterus is an abnormal finding and should be assessed further. Normal changes in the reproductive system related to aging include the graying and thinning of pubic hair, decreased size of the labia majora and clitoris, and loss of tone and elasticity of the pelvic ligaments and connective tissue. The uterus would normally be decreased, not increased, in size due to changes in hormonal levels and atrophy.

A 55-year-old male client is admitted to the emergency department with symptoms of a myocardial infarction. Which question by the nurse is the most appropriate before administering nitroglycerin? a. On a scale from 0 to 10, what is the rating of your chest pain? b. Are you allergic to any food or medications? c. Have you taken any drugs like Viagra recently? d. Are you light-headed or dizzy right now?

ANS: C Phosphodiesterase-5 inhibitors such as sildenafil (Viagra) relax smooth muscles to increase blood flow to the penis for treatment of erectile dysfunction. In combination with nitroglycerin, there can be extreme hypotension with reduction of blood flow to vital organs. The other questions are appropriate but not the highest priority before administering nitroglycerin.

A 25-year-old client has recently been diagnosed with testicular cancer and is scheduled for radiation therapy. Which intervention by the nurse is best? a. Ask the client about his support system of friends and relatives. b. Encourage the client to verbalize his fears about sexual performance. c. Explore with the client the possibility of sperm collection. d. Provide privacy to allow time for reflection about the treatment.

ANS: C Sperm collection is a viable option for a client diagnosed with testicular cancer and should be completed before The other options would promote radiation therapy, chemotherapy, or radical lymph node dissection. psychosocial support but are not the priority intervention.

A client is scheduled for a laparoscopy to remove endometriosis tissue. Which response by the client alerts the nurse of the need for further teaching? a. The surgeon told me that carbon dioxide would be infused into my pelvic cavity. b. There will be one or more small incisions in order to visualize all of the organs. c. There will be some shoulder pain after the procedure that may last 48 hours. d. I can return to jogging my 3-mile routine in a few days.

ANS: D No strenuous activity should occur for 7 days after the procedure. Carbon dioxide is infused into the pelvic cavity to visualize the organs. There are only one or more small incisions with this procedure. The referred shoulder pain that will occur should only last 48 hours.

A client is to have radiation therapy after a modified radical mastectomy. The nurse should teach the client to care for the skin at the site of therapy by: a. Washing the area with water. b. Exposing the area to dry heat. c. Applying an ointment to the area. d. Using talcum powder on the area

Ans: A A client receiving radiation therapy should avoid lotions, ointments, and anything that may cause irritation to the skin, such as exposure to sunlight, heat, or talcum powder. The area may safely be washed with water if it is done gently and if care is taken not to injure the skin.

The primary reason for taping an indwelling catheter laterally to the thigh of a male client is to: a. Eliminate pressure at the penoscrotal angle. b. Prevent the catheter from kinking in the urethra. c. Prevent accidental catheter removal. d. Allow the client to turn without kinking the catheter.

Ans: A The primary reason for taping an indwelling catheter to a male client so that the penis is held in a lateral position is to prevent pressure at the penoscrotal angle. Prolonged pressure at the penoscrotal angle can cause a ureterocutaneous fistula

The nurse is admitting a client diagnosed with Stage Ia cancer of the cervix to an outpatient surgery center for a conization. Which data would the client most likely report? a. Diffuse watery discharge. b. No symptoms. c. Dyspareunia. d. Intense itching.

Ans: B At this stage the client is asymptomatic and the cancer has been determined by a Pap smear.

In discussing home care with a client after transurethral resection of the prostate (TURP), the nurse should teach the male client that dribbling of urine: a. Can be a chronic problem. b. Can persist for several months. c. Is an abnormal sign that requires intervention. d. Is a sign of healing within the prostate.

Ans: B Dribbling of urine can occur for several months after TURP. The client should be informed that this is expected and is not an abnormal sign. The nurse should teach the client perineal exercises to strengthen sphincter tone. The client may need to use pads for temporary incontinence. The client should be reassured that continence will return in a few months and will not be a chronic problem. Dribbling is not a sign of healing but is related to the trauma of surgery.

The nurse is caring for a client diagnosed with uterine cancer who has been receiving systemic therapy for six (6) months. Which intervention should the nurse implement first? a. Determine which antineoplastic medication the client has received. b. Ask the client if she has had any problems with mouth ulcers at home. c. Administer the biologic response modifier filgrastim (Neupogen). d. Encourage the client to discuss feelings about having cancer.

Ans: B The systemic side effects of chemotherapy are not always apparent, and the development of stomatitis can be extremely distressing for the client. The nurse should assess the client's tolerance to treatments

When teaching a client about ovarian cancer, the nurse should include which of the following? Select all that apply. a. Details about the prognosis. b. Staging and grading of cancer. c. Need for routine colonoscopy beginning at age 30. d. Procedures for diagnosis if there is a pelvic mass. e. Symptoms occur early in the disease process

Ans: B, D Client teaching emphasizes the importance of regular gynecologic examinations. If a pelvic mass is found, completely explain the procedures for diagnosis. Explain pre-surgical and post-surgical instructions and explain the terminology particular to staging and grading of cancer, when appropriate. Refer all questions about the prognosis to the physician. Routine colonoscopies are typically begun at age 50 unless family history warrants otherwise. Ovarian tumors are commonly occult until symptoms of advanced disease are present.

The nurse and unlicensed assistive personnel (UAP) are caring for clients on a gynecology surgery floor. Which intervention cannot be delegated to the UAP? a. Empty the indwelling catheter on the three (3)-hour postoperative client. b. Assist the client who is two (2) days post hysterectomy to the bathroom. c. Monitor the peri-pad count on a client diagnosed with fibroid tumors. d. Encourage the client who is refusing to get out of bed to walk in the hall.

Ans: C . Monitoring a peri-pad count is done to determine if the client is bleeding excessively; the nurse should do this as part of the assessment.

A client has had a needle biopsy of the prostate gland using the transrectal approach. Which statement is most important to include in the client teaching plan? a. "The doctor will call you about the test results in a day or two. b. "Serious infections may occur as a complication of this test.' c. "You will need to call the doctor if you develop a fever or chills." d. "It is normal to have a small amount of rectal bleeding after the test."

Ans: C Although infection occurs only rarely as a complication of transrectal prostate biopsy, it is important that the client receive teaching about checking his temperature and calling the physician if there is any fever or other signs of systemic infection. The client should understand that the test results will not be available immediately but that he will be notified about the results. Transient rectal bleeding may occur after the biopsy, but bleeding that lasts for more than a few hours indicates that there may have been rectal trauma.

The charge nurse is making rounds on the genitourinary surgery floor. Which action by the primary nurse warrants immediate intervention? a. The nurse elevates the scrotum of a client who has had an orchiectomy. b. The nurse encourages the client to cough, although he complains of pain. c. The nurse empties the client's JP drain and leaves it rounded. d. The nurse asks the UAP to empty a catheter drainage bag.

Ans: C The Jackson Pratt (JP) drain is a drain attached to a bulb, and the bulb should remain compressed to apply gentle suction to the surgical site.

During the postoperative period after a modified radical mastectomy, the client confides in the nurse that she thinks she got breast cancer because she had an abortion and she did not tell her husband. The best response by the nurse is which of the following? a. "Cancer is not a punishment; it is a disease." b. "You might feel better if you confided in your husband." c. "Tell me more about your feelings on this." d. "I can have the social worker talk to you if you would like."

Ans: C The nurse should respond with an open-ended statement that elicits further exploration of the client's feelings. Women with cancer may feel guilt or shame. Previous life decisions, sexuality, and religious beliefs may influence a client's adjustment to a diagnosis of cancer. The nurse should not contradict the client's feelings of punishment or offer advice such as confiding in the husband. A social worker referral may be beneficial in the future but is not the first response needed to elicit exploration of the client's feelings.

A 67-year-old client with BPH has a new prescription for tamsulosin (Flomax). Which statement about tamsulosin is most important to include when teaching this client? a. "This medication will improve your symptoms by shrinking the prostate." b. "The force of your urinary stream will probably increase." c. "Your blood pressure will decrease as a result of taking this medication." d. "You should avoid sitting up or standing up too quickly."

Ans: D Because tamsulosin blocks alpha receptors in the peripheral arterial system, the most significant side effects are orthostatic hypotension and dizziness. To avoid falls, it is important that the client change positions slowly. The other information is also accurate and may be included in client teaching but is not as important as decreasing the risk for falls.

The nurse is caring for a client who will have a bilateral orchiectomy. The client asks what is involved with this procedure. The nurse's most appropriate response would be? "The surgery: a. Removes the entire prostate gland, prostatic capsule, and seminal vesicles." b. Tends to cause urinary incontinence and impotence." c. Freezes prostate tissue, killing cells." d. Results in reduction of the major circulating androgen, testosterone."

Ans: D Bilateral orchiectomy (removal of testes) results in reduction of the major circulating androgen, testosterone, as a palliative measure to reduce symptoms and progression of prostate cancer. A radical prostatectomy (removal of entire prostate gland, prostatic capsule, and seminal vesicles) may include pelvic lymphadenectomy. Complications include urinary incontinence, impotence, and rectal injury with the radical prostatectomy. Cryosurgery freezes prostate tissue, killing tumor cells without prostatectomy.

The client is diagnosed with early cancer of the prostate. Which assessment data would the client report? a. Urinary urgency and frequency. b. Retrograde ejaculation during intercourse. c. Low back and hip pain. d. No problems have been noticed

Ans: D In early-stage prostate cancer, the man will not be aware of the disease. Early detection is achieved by screening for the cancer.

A woman tells the nurse that "there's been a lot of cancer in my family." The nurse should instruct the client to report which possible sign of cervical cancer? a. Pain. b. Leg edema. c. Urinary and rectal symptoms. d. Light bleeding or watery vaginal discharge.

Ans: D In its early stages, cancer of the cervix is usually asymptomatic, which underscores the importance of regular Pap smears. A light bleeding or serosanguineous discharge may be apparent as the first noticeable symptom. Pain, leg edema, urinary and rectal symptoms, and weight loss are late signs of cervical cancer.

The client diagnosed with testicular cancer is scheduled for a unilateral orchiectomy. Which information is important to teach regarding sexual functioning? a. The client will have ejaculation difficulties after the surgery. b. The client will be prescribed male hormones following the surgery. c. The client may need to have a penile implant to be able to have intercourse. d. Libido and orgasm usually are unimpaired after this surgery

Ans: D Sex drive (libido) and orgasms usually are unimpaired because the client still has one testicle

A 34-year-old client comes to the clinic with concerns about an enlarged left testicle and heaviness in his lower abdomen. Which diagnostic test would the nurse expect to be ordered to confirm testicular cancer? a. Alpha-fetoprotein (AFP) b. Prostate-specific antigen (PSA) c. Prostate acid phosphatase (PAP) d. C-reactive protein (CRP)

ANS: A AFP is a glycoprotein that is elevated in testicular cancer. PSA and PAP testing is used in the screening of prostate cancer. CRP is diagnostic for inflammatory conditions.

The nurse is conducting a history on a male client to determine the severity of symptoms associated with prostate enlargement. Which finding is cause for prompt action by the nurse? a. Cloudy urine b. Urinary hesitancy c. Post-void dribbling d. Weak urinary stream

ANS: A Cloudy urine could indicate infection due to possible urine retention and should be a priority action. Common symptoms of benign prostatic hyperplasia are urinary hesitancy, post-void dribbling, and a weak urinary stream due to the enlarged prostate causing bladder outlet obstruction.

A client is placed on a medical regimen of doxorubicin (Adriamycin), cyclophosphamide (Cytoxan), fluorouracil (5-FU) for breast cancer. Which side effect seen in the client should the nurse report to the provider immediately? a. Shortness of breath b. Nausea and vomiting c. Hair loss d. Mucositis

ANS: A Doxorubicin (Adriamycin) can cause cardiac problems with symptoms of extreme fatigue, shortness of breath, chronic cough, and edema. These need to be reported as soon as possible to the provider. Nausea, vomiting. hair loss, and mucositis are common problems associated with chemotherapy regimens.

The nurse is teaching a 45-year-old woman about her fibrocystic breast condition. Which statement by the client indicates a lack of understanding? a. This condition will become malignant over time. b. I should refrain from using hormone replacement therapy. c. One cup of coffee in the morning should be enough for me. d. This condition makes it more difficult to examine my breasts.

ANS: A Fibrocystic breast condition does not increase a woman's chance of developing breast cancer. Hormone replacement therapy is not indicated since the additional estrogen may aggravate the condition. Limiting caffeine intake may give relief for tender breasts. The fibrocystic changes to the breasts make it more difficult to examine the breasts because of fibrotic changes and lumps

A client has scheduled brachytherapy sessions and states that she feels as though she is not safe around family. What is the best response by the nurse? a. You are only reactive when the radioactive implant is in place. b. To be totally safe, it's a good idea to sleep in a separate room c. It is best to stay a safe distance from friends or family between treatments. d. You should use a separate bathroom from the rest of the family.

ANS: A In brachytherapy, the surgeon inserts an applicator into the uterus. After placement is verified, the radioactive isotope is placed in the applicator for several minutes for a single treatment. There are no restrictions for the woman to stay away from her family or public between treatments

A client is diagnosed with metastatic prostate cancer. The client asks the nurse the purpose of his treatment with the luteinizing hormonereleasing hormone (LH-RH) agonist leuprolide (Lupron) and the bisphosphonate pamidronate (Aredia). Which statement by the nurse is most appropriate? a. The treatment reduces testosterone and prevents bone fractures. b. The medications prevent erectile dysfunction and increase libido. c. There is less gynecomastia and osteoporosis with this drug regimen. d. These medications both inhibit tumor progression by blocking androgens.

ANS: A Lupron, an LH-RH agonist, stimulates the pituitary gland to release luteinizing hormone (LH) to the point that the gland is depleted of LH and testosterone production is lessened. This may decrease the prostate cancer since it is hormone dependent. Lupron can cause osteoporosis, which results in the need for Aredia to prevent bone loss. Erectile dysfunction, decreased libido, and gynecomastia are side effects of the LH-RH medications. Antiandrogen drugs inhibit tumor progression by blocking androgens at the site of the prostate.

A nurse is caring for four postoperative clients who each had a total abdominal hysterectomy. Which client should the nurse assess first upon initial rounding? a. Client who has had two saturated perineal pads in the last 2 hours b. Client with a temperature of 99 F and blood pressure of 115/73 mm Hg c. Client who has pain of 4 on a scale of 0 to 10 d. Client with a urinary catheter output of 150 mL in the last 3 hours

ANS: A Normal vaginal bleeding should be less than one saturated perineal pad in 4 hours. Two saturated pads in such client a short time could indicate hemorrhage, which is a priority. The other clients also have needs, but the with excessive bleeding should be assessed first.

A woman diagnosed with breast cancer had these laboratory tests and performed at an office visit: Alkaline phosphatase 125 U/L Total calcium 12 mg/dL. Hematocrit 39% Hemoglobin 14 g/dL Which test results indicate to the nurse that some further diagnostics are needed? a. Elevated alkaline phosphatase and calcium suggests bone involvement. b. Only alkaline phosphatase is decreased, suggesting liver metastasis. c. Hematocrit and hemoglobin are decreased, indicating anemia. d. The elevated hematocrit and hemoglobin indicate dehydration.

ANS: A The alkaline phosphatase (normal value 30 to 120 U/L) and total calcium (normal value 9 to 10.5 mg/dL.) levels are both elevated, suggesting bone metastasis. Both the hematocrit and hemoglobin are within normal limits for females.

he nurse is teaching a client with benign prostatic hyperplasia (BPH). What statement indicates a lack of understanding by the client? a. There should be no problem with a glass of wine with dinner each night. b. I am so glad that I weaned myself off of coffee about a year ago. c. I need to inform my allergist that I cannot take my normal decongestant. d. My normal routine of drinking a quart of water during exercise needs to change.

ANS: A This client did not associate wine with the avoidance of alcohol, and requires additional teaching. The nurse must teach a client with BPH to avoid alcohol, caffeine, and large quantities of fluid in a short amount of time to prevent overdistention of the bladder. Decongestants also need to be avoided to lower the chance for urinary retention.

A client is discharged to home after a modified radical mastectomy with two drainage tubes. Which statement by the client would indicate that further teaching is needed? a. I am glad that these tubes will fall out at home when I finally shower. b. I should measure the drainage each day to make sure it is less than an ounce. c. I should be careful how I lie in bed so that I will not kink the tubing. d. If there is a foul odor from the drainage, I should contact my doctor.

ANS: A The drainage tubes (such as a Jackson-Pratt drain) lie just under the skin but need to be removed by the health care professional in about 1 to 3 weeks at an office visit. Drainage should be less than 25 mL in a days time. The client should be aware of her positioning to prevent kinking of the tubing. A foul odor from the drainage may indicate an infection; the doctor should be contacted immediately.

The nurse is giving discharge instructions to a client who had a total abdominal hysterectomy. Which statements by the client indicate a need for further teaching? (Select all that apply.) a. I should not have any problems driving to see my mother, who lives 3 hours away. b. Now that I have time off from work, I can return to my exercise routine next week. c. My granddaughter weighs 23 pounds, so I need to refrain from picking her up. d. I will have to limit the times that I climb our stairs at home to morning and night. e. For I month, I will need to refrain from sexual intercourse.

ANS: A, B Driving and sitting for extended periods of time should be avoided until the surgeon gives permission. For 2 to 6 weeks, exercise participation should also be avoided. All of the other responses demonstrate adequate knowledge for discharge. The client should not lift anything heavier than 10 pounds, should limit stair climbing, and should refrain from sexual intercourse.

A woman has been using acupuncture to treat the nausea and vomiting caused by the side effects of chemotherapy for breast cancer. Which conditions would cause the nurse to recommend against further use of acupuncture? (Select all that apply.) a. Lymphedema b. Bleeding tendencies c. Low white blood cell count d. Elevated serum calcium e. High platelet count

ANS: A, B, C Acupuncture could be unsafe for the client if there is poor drainage of the extremity with lymphedema or if there was a bleeding tendency and low white blood cell count. Coagulation would be compromised with a bleeding disorder, and the risk of infection would be high with the use of needles. An elevated serum calcium and high platelet count would not have any contraindication for acupuncture.

The nurse is taking the history of a 24-year-old client diagnosed with cervical cancer. What possible risk factors would the nurse assess? (Select all that apply.) a. Smoking b. Multiple sexual partners c. Poor diet d. Nulliparity e. Younger than 18 at first intercourse

ANS: A, B, C, E Smoking, multiple sexual partners, poor diet, and age less than 18 for first intercourse are all risk factors for cervical cancer. Nulliparity is a risk factor for endometrial cancer.

The nurse is assessing a client for reproductive health problems. What would be the priority assessments? (Select all that apply.) a. Bleeding b. Pain c. Sexual orientation d. Masses e. Discharge

ANS: A, B, D, E Bleeding, pain, masses, and discharge are common health problems that bring a client to a health care provider. Sexual orientation is not considered a health problem. Sexual activity should be assessed as part of the clients history.

The nurse is administering finasteride (Proscar) and doxazosin (Cardura) to a 67-year-old client with benign prostatic hyperplasia. What precautions are related to the side effects of these medications? (Select all that apply). a. Assessing for blood pressure changes when lying, sitting, and arising from the bed b. Immediately reporting any change in the alanine aminotransferase laboratory test c. Teaching the client about the possibility of increased libido with these medications d. Taking the clients pulse rate for a minute in anticipation of bradycardia e. Asking the client to report any weakness, light-headedness, or dizziness

ANS: A, B, E Both the 5-alpha-reductase inhibitor (5-ARI) and the alpha-selective blocking agents can cause orthostatic (postural) hypotension and liver dysfunction. The 5-ARI agent (Proscar) can cause a decreased libido rather than an increased sexual drive. The alpha-blocking drug (Cardura) can cause tachycardia rather than bradycardia.

The nurse is taking a history of a 68-year-old woman. What assessment findings would indicate a high risk for the development of breast cancer? (Select all that apply.) a. Age greater than 65 years b. Increased breast density c. Osteoporosis d. Multiparity e. Genetic factors

ANS: A, B, E The high risk factors for breast cancer are age greater than 65 with the risk increasing until age 80; an increase in breast density because of more glandular and connective tissue; and inherited mutations of BRCA1 and/or BRCA2 genes. Osteoporosis and multiparity are not risk factors for breast cancer. A high postmenopausal bone density and nulliparity are moderate and low increased risk factors, respectively.

A postmenopausal client is experiencing low back and pelvic pain, fatigue, and bloody vaginal discharge. What laboratory tests would the nurse expect to see ordered for this client if endometrial cancer is suspected? (Select all that apply.) a. Cancer antigen-125 (CA-125) b. White blood cell (WBC) count c. Hemoglobin and hematocrit (H&H) d. International normalized ratio (INR) e. Prothrombin time (PT)

ANS: A, C Serum tumor markers such as CA-125 assess for metastasis, especially if elevated. H&H would evaluate the possibility of anemia, a common finding with postmenopausal bleeding with endometrial cancer. WBC count is not indicated since there are no signs of infection. The INR and PT are coagulation tests to measure the time it takes for a fibrin clot to form. They are used to evaluate the extrinsic pathway of coagulation in clients receiving oral warfarin.

After a breast examination, the nurse is documenting assessment findings that indicate possible breast cancer. Which abnormal findings need to be included as part of the clients electronic medical record? (Select all that apply.) a. Peau dorange b. Dense breast tissue c. Nipple retraction d. Mobile mass at two oclock e. Nontender axillary nodes

ANS: A, C, D In the documentation of a breast mass, skin changes such as dimpling (peau dorange), nipple retraction, and whether the mass is fixed or movable are charted. The location of the mass should be stated by the face of a clock. Dense breast tissue and nontender axillary nodes are not abnormal assessment findings that may indicate breast cancer.

A 28-year-old client is diagnosed with endometriosis and is experiencing severe symptoms. Which actions by the nurse are the most appropriate at this time? (Select all that apply.) a. Reduce the pain by low-level heat. b. Discuss the high risk of infertility with this diagnosis. c. Relieve anxiety by relaxation techniques and education. d. Discuss in detail the side effects of laparoscopic surgery. e. Suggest resources such as the Endometriosis Association.

ANS: A, C, E With endometriosis, pain is the predominant symptom, with anxiety occurring because of the diagnosis. Interventions should be directed to pain and anxiety relief, such as low-level heat, relaxation techniques, and education about the pathophysiology and possible treatment of endometriosis. The nurse could suggest resources to give more information about the diagnosis. Discussion of the possibility of infertility and side effects of laparoscopic surgery is premature and may increase the anxiety.

A client is interested in learning about the risk factors for prostate cancer. Which factors does the nurse include in the teaching? (Select all that apply.) a. Family history of prostate cancer b. Smoking c. Obesity d. Advanced age e. Eating too much red meat f. Race

ANS: A, D, E, F Advanced family history of prostate cancer, age, a diet high in animal fat, and race are all risk factors for prostate cancer. Smoking and obesity are not known risk factors.

A 76-year-old client tells the nurse that she has lived long and does not need mammograms. Which is the nurse's best response? a. "Having a mammogram when you are older is less painful." b. "The incidence of breast cancer increases with age." c. "We need to consider your family history of breast cancer first." d. "It will be sufficient if you perform breast examinations monthly."

ANS: B Advancing age in postmenopausal women has been identified as a risk factor for breast cancer. A 76-year-old client needs monthly breast self-examination and a yearly clinical breast examination and mammogram to comply with the screening schedule. While mammograms are less painful as breast tissue becomes softer, the nurse should advise the woman to have the mammogram. Family history is important, but only about 5% of breast cancers are genetic.

The nurse is administering sulfamethoxazole-trimethoprim (Bactrim) to a client diagnosed with bacterial prostatitis. Which finding causes the nurse to question this medication for this client? a. Urinary tract infection b. Allergy to sulfa medications c. Hematuria d. Elevated serum white blood cells

ANS: B Before administering sulfamethoxazole-trimethoprim, the nurse must assess if the client is allergic to sulfa. Urinary tract infection, hematuria, and elevated serum white blood cells are common problems associated with bacterial prostatitis that require long-term antibiotic therapy.

A client has returned from a transurethral resection of the prostate with a continuous bladder irrigation. Which action by the nurse is a priority if bright red urinary drainage and clots are noted 5 hours after the surgery? a. Review the hemoglobin and hematocrit as ordered. b. Take vital signs and notify the surgeon immediately. c. Release the traction on the three-way catheter. d. Remind the client not to pull on the catheter.

ANS: B Bright red urinary drainage with clots may indicate arterial bleeding. Vital signs should be taken and the surgeon notified. The traction on the three-way catheter should not be released since it places pressure at the surgical site to avoid bleeding. The nurses review of hemoglobin and hematocrit and reminding the client not to pull on the catheter are good choices, but not the priority at this time.

A client is concerned about the risk of lymphedema after a mastectomy. Which response by the nurse is best? a. You do not need to worry about lymphedema since you did not have radiation therapy. b. A risk factor for lymphedema is infection, so wear gloves when gardening outside. c. Numbness, tingling, and swelling are common sensations after a mastectomy. d. The risk for lymphedema is a real threat and can be very self-limiting.

ANS: B Infection can create lymphedema; therefore, the client needs to be cautious with activities using the affected arm, such as gardening. Radiation therapy is just one of the factors that could cause lymphedema. Other risk factors include obesity and the presence of axillary disease. The symptoms of lymphedema are heaviness, aching, fatigue, numbness, tingling, and swelling, and are not common after the surgery. Women with lymphedema live fulfilling lives.

A client has recently been diagnosed with stage III endometrial cancer and asks the nurse for an explanation. What response by the nurse is correct about the staging of the cancer? a. The cancer has spread to the mucosa of the bowel and bladder. b. It has reached the vagina or lymph nodes. c. The cancer now involves the cervix. d. It is contained in the endometrium of the cervix.

ANS: B Stage III of endometrial cancer reaches the vagina or lymph nodes. Stage I is confined to the endometrium. Stage II involves the cervix, and stage IV spreads to the bowel or bladder mucosa and/or beyond the pelvis.

A nurse and unlicensed assistive personnel (UAP) are helping a client during a hysterosalpingogram. Which action by the nurse is best delegated to the UAP? a. Witnessing of the consent form b. Assisting the client into a lithotomy position c. Asking about allergies to iodine or shellfish d. Assessing for pelvic or shoulder pain after the study

ANS: B The UAP would be able to position the client for the procedure. Only the nurse has the ability to witness the consent form and assess allergies and pain within the nursing scope of practice.

A 68-year-old male client is embarrassed about having bilateral breast enlargement. Which statement by the nurse is the most appropriate? a. Breast cancer in men is quite rare. b. It is good that you came to be carefully evaluated. c. Gynecomastia usually comes from overeating. d. When you get older, the male breast always enlarges.

ANS: B The most appropriate statement is the one that is supportive of the client. A breast mass should be carefully evaluated for breast cancer, even if it is not common. Gynecomastia as a symptom can be related to antiandrogen agents, aging, obesity, estrogen excess, or lack of androgens.

The nurse is taking the history of a client who is scheduled for breast augmentation surgery. The client reveals that she took two aspirin this morning for a headache. Which action by nurse is best? a. Take the clients vital signs and record them in the chart. b. Notify the surgeon about the aspirin ingestion by the client. c. Warn the client that health insurance may not pay for the procedure. d. Teach the client about avoiding twisting above the waist after the operation.

ANS: B The surgeon must be notified immediately since the aspirin could cause increased bleeding during the procedure. Vital signs should be recorded and postoperative teaching should be completed in the preoperative time frame, but these are not the priority since the procedure may be rescheduled. The warning about the clients health insurance is not appropriate at this time.

A client is concerned about her irregular menstrual periods since she has increased her daily workouts at the gym to 2 hours each day. What is the nurses best response? a. Do you want to talk about the need for that much exercise? b. Exercise is healthy but can decrease body fat and cause irregular periods. c. Bingeing and purging can cause electrolyte problems in your body. d. Anorexic behavior can result in decreased estrogen levels.

ANS: B There needs to be a certain level of body fat and weight to maintain regular menstrual cycles. The client has only indicated that she has increased her workouts. There is no indication that she has anorexic or bingeing and purging behaviors.

A client is scheduled to start external beam radiation therapy (EBRT) for her endometrial cancer. Which teaching by the nurse is accurate? (Select all that apply.) a. You will need to be hospitalized during this therapy. b. Your skin needs to be inspected daily for any breakdown. c. It is not wise to stay out in the sun for long periods of time. d. The perineal area may become damaged with the radiation. e. The technician applies new site markings before each treatment.

ANS: B, C, D EBRT is usually performed in ambulatory care and does not require hospitalization. The client needs to know to evaluate the skin, especially in the perineal area, for any breakdown, and avoid sunbathing. The technician does not apply new site markings, so the client needs to avoid washing off the markings that indicate the treatment site.

A client has just returned from a right radical mastectomy. Which action by the unlicensed assistive personnel (UAP) would the nurse consider unsafe? a. Checking the amount of urine in the urine catheter collection bag b. Elevating the right arm on a pillow c. Taking the blood pressure on the right arm d. Encouraging the client to squeeze a rolled washcloth

ANS: C Health care professionals need to avoid the arm on the side of the surgery for blood pressure measurement, injections, or blood draws. Since lymph nodes are removed, lymph drainage would be compromised. The pressure from the blood pressure cuff could promote swelling. Infection could occur with injections and blood draws. Checking urine output, elevation of the affected arm on a pillow, and encouraging beginning exercises are all safe postoperative interventions

Which finding in a female client by the nurse would receive the highest priority of further diagnostics? a. Tender moveable masses throughout the breast tissue b. A 3-cm firm, defined mobile mass in the lower quadrant of the breast c. Nontender immobile mass in the upper outer quadrant of the breast d. Small, painful mass under warm reddened skin

ANS: C Malignant lesions are hard, nontender, and usually located in the upper outer quadrant of the breast and would be the priority for further diagnostic study. The other lesions are benign breast disorders. The tender moveable masses throughout the breast tissue could be a fibrocystic breast condition. A firm, defined mobile mass in the lower quadrant of the breast is a fibroadenoma, and a painful mass under warm reddened skin could be a local abscess or ductal ectasia.

With a history of breast cancer in the family, a 48-year-old female client is interested in learning about the modifiable risk factors for breast cancer. After the nurse explains this information, which statement made by the client indicates that more teaching is needed? a. I am fortunate that I breast-fed each of my three children for 12 months. b. It looks as though I need to start working out at the gym more often. c. I am glad that we can still have wine with every evening meal. d. When I have menopausal symptoms, I must avoid hormone replacement therapy.

ANS: C Modifiable risk factors can help prevent breast cancer. The client should lessen alcohol intake and not have wine 7 days a week. Breast-feeding, regular exercise, and avoiding hormone replacement are also strategies for breast cancer prevention

What comfort measure can only be performed by a nurse, as opposed to an unlicensed assistive personnel (UAP), for a client who returned from a left modified radical mastectomy 4 hours ago? a. Placing the head of bed at 30 degrees b. Elevating the left arm on a pillow c. Administering morphine for pain at a 4 on a 0-to-10 scale d. Supporting the left arm while initially ambulating the client

ANS: C Only the nurse is authorized to administer medications, but the UAP could inform the nurse about the rating of pain by the client. The UAP could position the bed to 30 degrees and elevate the clients arm on a pillow to facilitate lymphatic fluid drainage return. The clients arm should be supported while walking at first but then allowed to hang straight by the side. The UAP could support the arm while walking the client.

A 35-year-old woman is diagnosed with stage III breast cancer. She seems to be extremely anxious. What action by the nurse is best? a. Encourage the client to search the Internet for information tonight. b. Ask the client if sexuality has been a problem with her partner. c. Explore the idea of a referral to a breast cancer support group. d. Assess whether there has been any mental illness in her past.

ANS: C Support for the diagnosis would be best with a referral to a breast cancer support group. The Internet may be a good source of information, but the day of diagnosis would be too soon. The nurse could assess the frequency and satisfaction of sexual relations but should not assume that there is a problem in that area. Assessment of mental illness is not an appropriate action.

A client is starting hormonal therapy with tamoxifen (Nolvadex) to lower the risk for breast cancer. information needs to be explained by the nurse regarding the action of this drug? a. It blocks the release of luteinizing hormone. b. It interferes with cancer cell division. c. It selectively blocks estrogen in the breast. d. It inhibits DNA synthesis in rapidly dividing cells.

ANS: C Tamoxifen (Nolvadex) reduces the estrogen available to breast tumors to stop or prevent growth. This drug does not block the release of luteinizing hormone to prevent the ovaries from producing estrogen; leuprolide (Lupron) does this. Chemotherapy agents such as ixabepilone (Ixempra) interfere with cancer cell division, and doxorubicin (Adriamycin) inhibits DNA synthesis in susceptible cells.

The nurse is examining a woman's breast and notes multiple small mobile lumps. Which question would be the most appropriate for the nurse to ask? a. When was your last mammogram at the clinic? b. How many cans of caffeinated soda do you drink in a day? c. Do the small lumps seem to change with your menstrual period? d. Do you have a first-degree relative who has breast cancer?

ANS: C The most appropriate question would be one that relates to benign lesions that usually change in response to hormonal changes within a menstrual cycle. Reduction of caffeine in the diet has been shown to give relief in fibrocystic breast conditions, but research has not found that it has a significant impact. Questions related to the clients last mammogram or breast cancer history are not related to the nurses assessment.

A 55-year-old African-American client is having a visit with his health care provider. What test should the nurse discuss with the client as an option to screen for prostate cancer, even though screening is not routinely recommended? a. Complete blood count b. Culture and sensitivity c. Prostate-specific antigen d. Cystoscopy

ANS: C The prostate-specific antigen test should be discussed as an option for prostate cancer screening. A complete blood count and culture and sensitivity laboratory test will be ordered if infection is suspected. A cystoscopy would be performed to assess the effect of a bladder neck obstruction.

The nurse is assessing the reproductive history of a 68-year-old postmenopausal woman. Which finding is cause for immediate action by the nurse? a. Vaginal dryness b. Need for a Papanicolaou test if none for 3 years c. Bleeding from the vagina d. Leakage of urine

ANS: C Vaginal bleeding is not normal for the postmenopausal woman. Vaginal dryness and leakage of urine are common findings in adults of this age range. Pap tests may not be needed for women over 65 who have had regular cervical cancer testing with normal results.

The nurse is reviewing discharge plans with a client who is recovering from a cervical biopsy. Which statements indicate good understanding by the client? (Select all that apply.) a. I can return to work this afternoon. b. There should be no problem lifting my 2-year-old toddler when I get home. c. I cannot douche until the biopsy site is healed. d. I need to wait for about 2 weeks to have intercourse with my husband. e. If I have some bleeding, I can use a regular tampon this evening.

ANS: C, D The client should not douche, have intercourse, or use tampons until the biopsy site is healed. The client should rest for 24 hours after the procedure and should not lift heavy objects.

The nurse is teaching a client who is undergoing brachytherapy about what to immediately report to her health care provider. Which signs and symptoms would be included in this teaching? (Select all that apply.) a. Constipation for 3 days b. Temperature of 99 F c. Abdominal pain d. Visible blood in the urine e. Heavy vaginal bleeding

ANS: C, D, E Health teaching for a client having brachytherapy should emphasize reporting abdominal pain, visible blood in the urine, and heavy vaginal bleeding. Severe diarrhea (not constipation), urethral burning, extreme fatigue, and a fever over 100 F should also be reported.

During dressing changes, the nurse assesses a client who has had breast reconstruction. Which finding would cause the nurse to take immediate action? a. Slightly reddened incisional area b. Blood pressure of 128/75 mm Hg c. Temperature of 99 F (37.2 C) d. Dusky color of the flap

ANS: D A dusky color of the breast flap could indicate poor tissue perfusion and a decreased capillary refill. The nurse should notify the surgeon to preserve the tissue. It is normal to have a slightly reddened incision as the skin heals. The blood pressure is within normal limits and the temperature is slightly elevated but should be monitored.

A client is diagnosed with a fibrocystic breast condition while in the hospital and is experiencing breast discomfort. What comfort measure would the nurse delegate to the unlicensed assistive personnel (UAP)? a. Aid in the draining of the cysts by needle aspiration. b. Teach the client to wear a supportive bra to bed. c. Administer diuretics to decrease breast swelling. d. Obtain a cold pack to temporarily relieve the pain.

ANS: D All of the options would be comfort measures for a client with a fibrocystic breast condition. The UAP can obtain the cold or heat therapy. Only the nurse should aid the health care provider with a needle aspiration, teach, and administer medications.

The client is emotionally upset about the recent diagnosis of stage IV endometrial cancer. Which action by the nurse is best? a. Let the client alone for a long period of reflection time. b. Ask friends and relatives to limit their visits. c. Tell the client that an emotional response is unacceptable. d. Create an atmosphere of acceptance and discussion.

ANS: D Discussion of a clients concerns about the presence of cancer and the potential for recurrence will provide emotional support and allay fears. Coping behaviors are encouraged with the support of friends and relatives. An emotional response should be accepted.

A client is diagnosed with benign prostatic hyperplasia and seems sad and irritable. After assessing the clients behavior, which statement by the nurse would be the most appropriate? a. The urine incontinence should not prevent you from socializing. b. You seem depressed and should seek more pleasant things to do. c. It is common for men at your age to have changes in mood. d. Nocturia could cause interruption of your sleep and cause changes in mood.

ANS: D Frequent visits to the bathroom during the night could cause sleep interruptions and affect the clients mood and mental status. Incontinence could cause the client to feel embarrassment and cause him to limit his activities outside the home. The social isolation could lead to clinical depression and should be treated professionally. The nurse should not give advice before exploring the clients response to his change in behavior. The statement about age has no validity.

The nurse is developing a teaching plan for a client who is scheduled for her first Papanicolaou test. What instruction by the nurse is the most accurate? a. The timing of the Pap smear does not matter. b. Sexual intercourse will not interfere with the results. c. Results can be interpreted immediately in the office. d. Results are best if you do not douche 24 hours before the test.

ANS: D In order to prevent false interpretation, the client must not douche or have sexual intercourse for at least 24 hours before the Pap smear. Timing is important, with the test scheduled between the clients menstrual periods so that the menstrual flow does not interfere with laboratory analysis. The specimens are placed on a glass slide and sent to the laboratory for examination and cannot be interpreted immediately.

The mother of an 18-year-old girl asks the nurse which screening her daughter should receive now based on evidence-based recommendations. Which suggestion by the nurse is best? a. Papanicolaou test b. Human papilloma virus (HPV) test c. Mammogram d. No screenings at this time

ANS: D Since the daughter is only 18, it is not recommended that she receive any of these screenings. Pap screenings are recommended to start at age 21. The HPV test can be done with the Pap test for women older than 30 or who had an abnormal Pap test result. A mammogram is recommended for women age 40 or older since cancers are more able to be distinguished from normal glandular tissue at that age.

A nurse and an unlicensed assistive personnel (UAP) are caring for a client with an open radical prostatectomy. Which comfort measure could the nurse delegate to the UAP? a. Administering an antispasmodic for bladder spasms b. Managing pain through patient-controlled analgesia c. Applying ice to a swollen scrotum and penis d. Helping the client transfer from the bed to the chair

ANS: D The UAP could aid the client in transferring from the bed to the chair and with ambulation. The nurse would be responsible for medication administration, assessment of swelling, and the application of ice if needed.

A client has just returned from a total abdominal hysterectomy and needs postoperative nursing care. What action can the nurse delegate to the unlicensed assistant personnel (UAP)? a. Assess heart, lung, and bowel sounds. b. Check the hemoglobin and hematocrit levels. c. Evaluate the dressing for drainage. d. Empty the urine from the catheter bag

ANS: D The UAP is able to empty the urinary output from the catheter. The nurse would assess the heart, lung, and bowel sounds; check the hemoglobin and hematocrit levels; and evaluate the drainage on the dressing.

A 37-year-old Nigerian woman is at high risk for breast cancer and is considering a prophylactic mastectomy and oophorectomy. What action by the nurse is most appropriate? a. Discourage this surgery since the woman is still of childbearing age. b. Reassure the client that reconstructive surgery is as easy as breast augmentation. c. Inform the client that this surgery removes all mammary tissue and cancer risk. d. Include support people, such as the male partner, in the decision making.

ANS: D The cultural aspects of decision making need to be considered. In the Nigerian culture, the man often makes the decisions for care of the female. Women with a high risk for breast cancer can consider prophylactic surgery. If reconstructive surgery is considered, the procedure is more complex and will have more complications compared to a breast augmentation. There is a small risk that breast cancer can still develop in the remaining mammary tissue.

A client has undergone a vaginal hysterectomy with a bilateral salpingo-oophorectomy. She is concerned about a loss of libido. What intervention by the nurse would be best? a. Suggest increasing vitamins and supplements daily. b. Discuss the value of a balanced diet and exercise. c. Reinforce that weight gain may be inevitable. d. Teach that estrogen cream inserted vaginally may help.

ANS: D Use of vaginal estrogen cream and gentle dilation can help with vaginal changes and loss of libido. Weight gain and masculinization are misperceptions after a vaginal hysterectomy. Vitamins, supplements, a balanced diet, and exercise are helpful for healthy living, but are not necessarily going to increase libido.

A 67-year-old male client had some serum tests performed during his annual examination. The nurse reviews his results, as follows: testosterone: 680 ng/dL; prostate-specific antigen: 10 ng/mL; prolactin: 5ng/mL. What action by the nurse is best? a. Assess for possible galactorrhea with breast discharge. b. Note the possibility of a testicular tumor. c. Communicate to the provider that results were normal. d. Prepare the client for further diagnostic testing.

ANS: D The prostate-specific antigen is increased from the normal of 0 to 2.5, which could indicate benign prostatic hyperplasia or prostate cancer. Further testing would have to be done. The values of testosterone and prolactin are within normal range. If the prolactin were increased, there would be a possibility of galactorrhea. An increase in testosterone could indicate a possible testicular tumor.

The client who is scheduled to have a breast biopsy with sentinel node dissection states, "I don't understand. What does a sentinel node biopsy do?" Which scientific rationale should the nurse use to base the response? a. A dye is injected into the tumor and traced to determine spread of cells. b. The surgeon removes the nodes that drain the diseased portion of the breast. c. The nodes felt manually will be removed and sent to pathology. d. A visual inspection of the lymph nodes will be made while the client is sleeping

Ans: A A sentinel node biopsy is a procedure in which a radioactive dye is injected into the tumor and then traced by instrumentation and color to try to identify the exact lymph nodes the tumor could have shed into.

A client asks the nurse, "Where is cancer usually found in the breast?" When responding to the client, the nurse uses a diagram of a left breast and indicates that most malignant tumors occur in which quadrant of the breast? a. Upper outer quadrant. b. Upper inner quadrant. c. Lower outer quadrant. d. Lower inner quadrant.

Ans: A About half of malignant breast tumors occur in the upper outer quadrant of the breast. For no known reason, cancer appears in the left breast more often than in the right breast. The upper outer quadrants of the breast, and especially the axillary area, should be covered thoroughly in the clinical breast examination and breast self-examination

After undergoing a modified radical mastectomy, a client is transferred to the postanesthesia care unit (PACU). Which nursing action is best to delegate to an experienced LPN/LVN? a. Monitoring the client's dressing for any signs of bleeding b. Documenting the initial assessment on the client's chart c. Communicating the client's status report to the charge nurse on the surgical unit d. Teaching the client about the importance of using pain medication as needed

Ans: A An LP/LVN working in a PACU would be expected to check dressings for bleeding and alert RN staff members if bleeding occurs. The other tasks are more appropriate for nursing staff with RN-level education and licensure.

During the admission workup for a modified radical mastectomy, the client is extremely anxious and asks many questions. Which of the following approaches would offer the best guide for the nurse to answer questions raised by this apprehensive preoperative client? a. Tell the client as much as she wants to know and is able to understand. b. Delay discussing the client's questions with her until she is convalescing. c. Delay discussing the client's questions with her until her apprehension subsides. d. Explain to the client that she should discuss her questions first with the physician.

Ans: A An important nursing responsibility is preoperative teaching, and the most frequently recommended guide for teaching is to tell the client as much as she wants to know and is able to understand. Delaying discussion of issues about which the client has concerns is likely to aggravate the situation and cause the client to feel distrust. As a general guide, the client would not ask the question if she were not ready to discuss her situation. The nurse is available to answer the client's questions and concerns and should not delay discussing these with the client.

Which could be a complication of cryotherapy surgery for cancer of the prostate? a. The urethra could become scarred and cause retention. b. The client could have ejaculation difficulties and be impotent. c. Bone marrow suppression could occur from the chemotherapy. d. Chronic vomiting and diarrhea causing electrolyte imbalance could occur.

Ans: A Cryotherapy involves placing freezing probes into the prostate to freeze the cancer cells. An indwelling catheter is placed into the urethra, and warm water is circulated through the catheter to try to prevent the urethra from freezing. If the urethra scars, then the lumen will constrict, causing retention of urine

The client has undergone a bilateral orchiectomy for cancer of the prostate. Which intervention should the nurse implement? a. Support the scrotal sac with a towel and apply ice. b. Administer testosterone replacement hormone orally. c. Encourage the client to place sperm in a sperm bank. d. Have the client talk to another man with ejaculation dysfunction

Ans: A Elevating a surgical site and applying ice will reduce edema to the area.

A client undergoing chemotherapy after a modified radical mastectomy asks the nurse questions about a breast prosthesis and wigs. After answering the questions directly, the nurse should also: a. Provide a list of resources, including the local breast cancer support group. b. Offer a referral to the social worker. c. Call the home health care agency. d. Contact the plastic surgeon

Ans: A Giving the client a list of community resources that could provide support and guidance assists the client to maintain her self-image and independence. The support group will include other women who have undergone similar therapies and can offer suggestions for breast products and wigs. Because the client is asking about specific resources, she does not need a referral to a social worker, home health agency, or plastic surgeon

After the surgeon's meeting with a client to obtain the client's informed consent for a modified radical mastectomy, the client asks the nurse many questions about breast reconstruction that the nurse finds difficult to answer. The nurse should: a. Inform the surgeon that the client has questions about reconstruction before she signs the consent. b. Inform the client that she should concentrate on recovering from the mastectomy first. c. Inform the client that she can have a consultation with the plastic surgeon in a few weeks. d. Inform the client she can ask the surgeon these questions later when the surgeon makes rounds

Ans: A If a client has questioned the nurse cannot answer, it is best to delay the signing of the consent until the questions are clarified for the client. The surgeon should be notified, and the appropriate information or collaboration should be provided for the client before she signs the surgical consent. Telling her she should concentrate on recovery first ignores the client's questions and concerns. Frequently the plastic surgeon needs to be consulted at the beginning of the treatment because various surgical decisions depend on the future plans for breast reconstruction

The nurse is performing a digital rectal examination. Which of the following finding is a key sign for prostate cancer? a. A hard prostate, localized or diffuse. b. Abdominal pain. c. A boggy, tender prostate. d. A nonindurated prostate

Ans: A On digital rectal examination, key signs of prostate cancer are a hard prostate, induration of the prostate, and an irregular, hard nodule. Accompanying symptoms of prostate cancer can include constipation, weight loss, and lymphadenopathy. Abdominal pain usually does not accompany prostate cancer. A boggy, tender prostate is found with infection (e.g., acute or chronic prostatitis).

The nurse is providing a community presentation about ovarian cancer. Which topic should receive priority attention in the lesson plan? a. Ovarian cancer signs and symptoms are often vague until late in development. b. Ovarian cancer should be considered in any woman older than 30 years of age. c. A rigid board-like abdomen is the most common sign. d. Methods for early detection have made a dramatic reduction in the mortality rate due to ovarian cancer.

Ans: A Ovarian cancer is rarely diagnosed early. Methods for mass screening and early detection have not been successful. Signs and symptoms are often vague until late in development. Ovarian cancer should be considered in any woman older than 40 years of age who has complaints of vague abdominal and/or pelvic discomfort or enlargement, a sense of bloating, or flatulence. Enlargement of the abdomen due to the accumulation of fluid is the most common sign

After a radical prostatectomy, a client is ready to be discharged. Which nursing action included in the discharge plan should be delegated to an experienced LPN/LVN? a. Reinforcing the client's need to check his temperature daily b. Teaching the client how to care for his retention catheter c. Documenting a discharge assessment in the client's chart d. Instructing the client about prescribed narcotic analgesics

Ans: A Reinforcement of previous teaching is an expected role of the LPN/LVN. Planning and implementing client initial teaching and documentation of a client's discharge assessment should be performed by experienced RN staff members.

The nurse is preparing an educational presentation for women in the community. Which primary nursing intervention should the nurse discuss regarding the development of ovarian cancer? a. Instruct the clients not to use talcum powder on the perineum. b. Encourage the clients to consume diets with a high-fat content. c. Teach the women to have a lower pelvic sonogram yearly. d. Discuss the need to be aware of the family history of cancer.

Ans: A Research has shown the use of talcum powder perineally increases the risk for developing ovarian cancer, although there is no explanation known for this occurrence. Other risk factors include a high fat diet, nulliparity, infertility, older age (70 to 80 years) has the greatest incidence, mumps before menarche, and family history of ovarian cancer.

The client in the gynecology clinic asks the nurse, "What are the risk factors for developing cancer of the cervix?" Which statement is the nurse's best response? a. "The earlier the age of sexual activity and the more partners, the greater the risk." b. "Eating fast foods high in fat and taking birth control pills are risk factors." c. "A Chlamydia trachomatis infection can cause cancer of the cervix." d. "Having yearly Pap smears will protect you from developing cancer."

Ans: A Risk factors for cancer of the cervix include sexual activity before the age of 20 years; multiple sexual partners; early childbearing; exposure to the human papillomavirus; HIV infection; smoking; and nutritional deficits of folates, beta carotene, and vitamin C.

Which is the American Cancer Society's recommendation for the early detection of cancer of the prostate? a. A yearly PSA level and DRE beginning at age 50. b. A biannual rectal examination beginning at age 40. c. A semiannual alkaline phosphatase level beginning at age 45. d. A yearly urinalysis to determine the presence of prostatic fluid.

Ans: A The American Cancer Society recommends all men have a yearly prostate-specific antigen (PSA) blood level, followed by a digital rectal examination (DRE) beginning at age 50. Men in the high-risk group, including all African American men, should begin at age 45

The nurse refers a client who had a mastectomy to "Reach to Recovery." The primary purpose of the American Cancer Society's Reach to Recovery program is to: a. Foster rehabilitation in women who have had mastectomies. b. Raise funds to support early breast cancer detection programs. c. Provide free dressings for women who have had radical mastectomies. d. Collect statistics for research from women who have had mastectomies

Ans: A The American Cancer Society's Reach to Recovery is a rehabilitation program for women who have had breast surgery. It is designed to meet their physical, psychological, and emotional needs. The Reach to Recovery program is implemented by women who have had breast cancer themselves. Many women benefit from this peer information and support.

A 72-year-old male is in the emergency department because he has been unable to void for the past 12 hours. The best method for the nurse to use when assessing for bladder distention in a male client is to check for: a. A rounded swelling above the pubis. b. Dullness in the lower left quadrant. c. Rebound tenderness below the symphysis. d. Urine discharge from the urethral meatus.

Ans: A The best way to assess for a distended bladder in either a male or female client is to check for a rounded swelling above the pubis. This swelling represents the distended bladder rising above the pubis into the abdominal cavity. Dullness does not indicate a distended bladder. The client might experience tenderness or pressure above the symphysis. No urine discharge is expected, the urine flow is blocked by the enlarged prostate.

The nurse is caring for a client newly diagnosed with Stage IV ovarian cancer. What is the scientific rationale for detecting the tumors at this stage? a. The client's ovaries lie deep within the pelvis and early symptoms are vague. b. The client has regular gynecological examinations and this helps with detection. c. The client had a history of dysmenorrhea and benign ovarian cysts. d. The client had a family history of breast cancer and was being checked regularly.

Ans: A The ovaries are anatomically positioned deep within the pelvis, and because of this, signs and symptoms of cancer are vague and nonspecific. Symptoms include increased abdominal girth, pelvic pressure, indigestion, bloating, flatulence, and pelvic and leg pain. Increasing abdomen size as a result of accumulation of fluid is the most common sign. Many women ignore the symptoms because they are so nonspecific.

A postmenopausal woman is worried about pain in the upper outer quadrant of her left breast. The nurse's first course of action is to: a. Do a breast examination and report the results to the physician. b. Explain that pain is caused by hormonal fluctuations. c. Reassure the client that pain is not a symptom of breast cancer. d. Teach the client the correct procedure for breast self-examination (BSE)

Ans: A This complaint warrants the nurse's performing an examination and reporting the results to the physician. Hormone fluctuations do cause breast discomfort, but an examination must be done at this time to assess the breast. Although pain is not common with breast cancer, it can be a symptom. Teaching the client to perform BSE is important, but it is not the priority action in this case.

The client who has had a mastectomy tells the nurse, "My husband will leave me now since I am not a whole woman anymore." Which response by the nurse is most therapeutic? a. "You're afraid your husband will not find you sexually appealing?" b. "Your husband should be grateful you will be able to live and be with him." c. "Maybe your husband would like to attend a support group for spouses." d. "You don't know that is true. You need to give him a chance."

Ans: A This is restating the client's feelings and is a therapeutic response

What instructions should the nurse provide to a client who develops cellulitis in the right arm after a right modified radical mastectomy? a. Antibiotics will need to be taken for 1 to 2 weeks. b. Arm exercises will get rid of the cellulitis. c. Ice pack should be applied to the affected area for 20-minute periods to reduce swelling. d. The right extremity should be lowered

Ans: A Treatment for cellulitis includes oral or intravenous antibiotics for 1 to 2 weeks, elevation of the affected extremity, and application of warm, moist packs to the site. Arm exercises help to reduce swelling, but do not treat the infection

The husband of a client with cervical cancer says to the nurse, "The doctor told my wife that her cancer is curable. Is he just trying to make us feel better?" Which would be the nurse's most accurate response? a. "When cervical cancer is detected early and treated aggressively, the cure rate is almost 100%." b. "The 5-year survival rate is about 75%, which makes the odds pretty good." c. "Saying a cancer is curable means that 50% of all women with the cancer survive at least 5 years." d. "Cancers of the female reproductive tract tend to be slow-growing and respond well to treatment."

Ans: A When cervical cancer is detected early and treated aggressively, the cure rate approaches 100%. The incidence of cervical cancer has increased among African Americans, Native Americans, and Latinas, and these women often have a poorer prognosis because the cancer is not identified early. Papanicolaou smears and colposcopy have the potential to decrease mortality from invasive carcinoma when these screening and treatment programs are utilized by women.

The nurse is teaching a client newly diagnosed with prostate cancer. Which of the following points should be included in the instruction? Select all that apply. a. Prostate cancer is usually multifocal and slow-growing. b. Most prostate cancers are adenocarcinoma. c. The incidence of prostate cancer is higher in African American men, and the onset is earlier. d. A prostate specific antigen (PSA) lab test greater than 4 ng/mg will need to be monitored. e. Cancer cells are detectable in the urine.

Ans: A, B, C, D Cancer of the prostate gland is the second-leading cause of cancer death among American men and is the most common carcinoma in men older than age 65. Incidence of prostate cancer is higher in African American men, and onset is earlier. Most prostate cancers are adenocarcinoma. Prostate cancer is usually multifocal, slow-growing, and can spread by local extension, by lymphatics, or through the bloodstream. Prostate-specific antigen (PSA) greater than 4 ng/mg is diagnostic; a free PSA level can help stratify the risk of elevated PSA levels. Metastatic workup may include skeletal x-ray, bone scan, and CT or MRI to detect local extension, bone, and lymph node involvement. The urine does not have prostate cancer cells.

The client diagnosed with cancer of the uterus is scheduled to have radiation brachytherapy. Which precautions should the nurse implement? Select all that apply. a. Place the client in a private room. b. Wear a dosimeter when entering the room. c. Encourage visitors to come and stay with the client. d. Plan to spend extended time with the client. e. Notify the nuclear medicine technician

Ans: A, B, E Brachytherapy is the direct implantation of radioactive seeds through the vagina into the uterus. The client should be in a private room at the end of the hall to prevent radiation exposure to the rest of the unit. Nurses wear a dosimeter registering the amount of radiation they have been exposed to. When a certain level is reached, the nurse is no longer allowed to care for clients undergoing internal radiation therapy. The nuclear medicine technician will assist with the placement of the implants and will deliver the implants in a lead lined container. The technician will also scan any items (linens and wastes) leaving the room for radiation contamination

The nurse is caring for a client who is one (1) day postoperative hysterectomy for cancer of the ovary. Which nursing interventions should the nurse implement? Select all that apply. a. Assess for calf enlargement and tenderness. b. Turn, cough, and deep breathe every six (6) hours. c. Assess pain on a 1-to-10 pain scale. d. Apply sequential compression devices to legs. e. Assess bowel sounds every four (4) hours.

Ans: A, C, D, E All clients who have had surgery are at risk for developing deep vein thrombosis (DVT), and an enlarged, tender calf is a sign of DVT. Clients who have had surgery should be assessed for pain on a pain scale and by observing for physiological markers indicating pain. Sequential compression hose is used prophylactically to prevent deep vein thrombosis. The client should be assessed for the return of bowel sounds.

A client develops lymphedema after a left mastectomy with lymph node dissection. Which of the following should be included in the discharge teaching plan? Select all that apply. a. Do not allow blood pressures or blood draws in the affected arm. b. Avoid application of sunscreen on the left arm. c. Use an electric razor for shaving. d. Immobilize the left arm. e. Elevate the left arm. f. Perform hand pump exercises.

Ans: A, C, E, F Blood pressures or blood draws in the affected arm, sun exposure, trauma with a sharp razor, and immobilization increase the risk of lymphedema. Elevation of the arm and hand pump exercises promote lymph flow and reduce edema.

You are working with a UAP to care for a client who has had a right breast lumpectomy and axillary lymph node dissection. Which nursing action can you delegate to the UAP? a. Teaching the client why blood pressure measurements are taken on the left arm b. Elevating the client's arm on two pillows to promote lymphatic drainage c. Assessing the client's right arm for lymphedema d. Reinforcing the dressing if it becomes saturated

Ans: B Positioning the client's arm is a task within the scope of practice for UAP working on a surgical unit. Client teaching and assessment are RN-level skills. The RN should reinforce dressings as necessary, because this requires assessment of the surgical site and possible communication with the surgeon.

The female client has a mother who died from ovarian cancer and a sister diagnosed with ovarian cancer. Which recommendations should the nurse make regarding early detection of ovarian cancer? a. The client should consider having a prophylactic bilateral oophorectomy. b. The client should have a transvaginal ultrasound and a CA-125 laboratory test every six (6) months. c. The client should have yearly magnetic resonance imaging (MRI) scans. d. The client should have a biannual gynecological examination with flexible sigmoidoscopy

Ans: B A transvaginal ultrasound is a sonogram in which the sonogram probe is inserted into the vagina and sound waves are directed toward the ovaries. The CA-125 tumor marker is elevated in several cancers. It is nonspecific but, coupled with the sonogram, can provide information about ovarian cancer for early diagnosis.

You are caring for a 21-year-old client who had a left orchiectomy for testicular cancer on the previous day. Which nursing activity will you delegate to an LPN/LVN? a. Educating the client about post-orchiectomy chemotherapy and radiation b. Administering the prescribed "as needed" (PRN) oxycodone (Roxicodone) to the client c. Teaching the client how to perform testicular self-examination on the remaining testicle d. Assessing the client's knowledge level about post-orchiectomy fertility

Ans: B Administration of narcotics and the associated client monitoring are included in LP/LVN education and scope of practice. Assessments and teaching are more complex skills that require RN-level education and are best accomplished by an RN with experience in caring for clients with this diagnosis.

The client states that she has noticed that her bra fits more snugly at certain times of the month. She asks the nurse if this is a sign of breast disease. The nurse should base the reply to this client on the knowledge that: a. Benign cysts tend to cause the breasts to vary in size. b. It is normal for the breasts to increase in size before menstruation begins. c. A change in breast size warrants further investigation. d. Differences in breast size are related to normal growth and development.

Ans: B Breasts may vary in size before menstruation because of breast engorgement caused by hormonal changes. A woman may then note that her bra fits more tightly than usual. Benign cysts do not cause variation in breast size. A change in breast size that does not follow hormonal changes could warrant further assessment. The breasts normally are about the same size, although some women have one breast slightly larger than the other

A nurse is assessing a female who is receiving her second administration of chemotherapy for breast cancer. When obtaining this client's health history, what is the most important information the nurse should obtain? a. "Has your hair been falling out in clumps?" b. "Have you had nausea or vomiting?" c. "Have you been sleeping at night?" d. "Do you have your usual energy level?"

Ans: B Chemotherapy agents typically cause nausea and vomiting when not controlled by antiemetic drugs. Antineoplastic drugs attack rapidly growing normal cells, such as in the gastrointestinal tract. These drugs also stimulate the vomiting center in the brain. Hair loss, loss of energy, and sleep are important aspects of the health history, but not as critical as the potential for dehydration and electrolyte imbalance caused by nausea and vomiting

You are caring for a client who has just returned to the surgical unit after a TURP. Which assessment finding will require the most immediate action? a. Blood pressure reading of 153/88 mm Hg b. Catheter that is draining deep red blood c. Client not wearing antiembolism hose d. Client reports of abdominal cramping

Ans: B Hemorrhage is a major complication after TURP and should be reported to the surgeon immediately. The other assessment data also indicate a need for nursing action, but not as urgently.

The primary reason for lubricating the urinary catheter generously before inserting it into a male client is that this technique helps reduce: a. Spasms at the orifice of the bladder. b. Friction along the urethra when the catheter is being inserted. c. The number of organisms gaining entrance to the bladder. d. The formation of encrustations that may occur at the end of the catheter.

Ans: B Liberal lubrication of the catheter before catheterization of a male reduces friction along the urethra and irritation and trauma to urethral tissues. Because the male urethra is tortuous, a liberal amount of lubrication is advised to ease catheter passage. The female urethra is not tortuous, and, although the catheter should be lubricated before insertion, less lubricant is necessary. Lubrication of the catheter will not decrease spasms. The nurse should use sterile technique to prevent introducing organisms. Crusts will not form immediately. Irrigating the catheter as needed will prevent clot and crust formation.

The school nurse is preparing a class on testicular cancer for male high school seniors. Which information regarding testicular self-examination should the nurse include? a. Perform the examination in a cool room under a fan. b. Any lump should be examined by an HCP as soon as possible. c. Discuss having a second person confirm a negative result. d. The procedure will cause mild discomfort if done correctly.

Ans: B The client may note a cordlike structure; this is the spermatic cord and is normal. Any lump or mass felt is abnormal and should be checked by an HCP as soon as possible.

The nurse is developing a program about prostate cancer for a health fair. The nurse should provide information about which of the following topics? a. The Prostate-Specific Antigen (PSA) test is reliable for detecting the presence of prostate cancer. b. For all men, age 50 and older, the American Cancer Society recommends an annual rectal examination. c. Avoid lifting more than 20 lb aids in prevention of prostate cancer. d. Regular sexual activity promotes health of the prostate gland to prevent cancer

Ans: B Most prostate cancer is adenocarcinoma and is palpable on rectal examination because it arises from the posterior portion of the gland. Although the PSA is not a perfect screening test, the American Cancer Society recommends an annual rectal examination and blood PSA level for all men age 50 and older or starting at age 40 if African American or if there is family history of prostate cancer. To help achieve optimal sexual function, give the client the opportunity to communicate his concerns and sexual needs. Regular sexual activity does not prevent cancer

The client has had a total abdominal hysterectomy for cancer of the uterus. Which discharge instruction should the nurse teach? a. The client should take HRT every day to prevent bone loss. b. The client should practice pelvic rest until seen by the HCP. c. The client can drive a car as soon as she is discharged from the hospital. d. The client should expect some bleeding after this procedure

Ans: B Pelvic rest means nothing is placed in the vagina. The client does not need a tampon at this time, but sexual intercourse should be avoided until the vaginal area has healed

A 36-year-old female is scheduled to receive external radiation therapy and a cesium implant for cancer of the cervix. Which of the following statements would be most accurate to include in the teaching plan about the potential effects of radiation therapy on sexuality? a. "You can have sexual intercourse while the implant is in place." b. "You may notice some vaginal dryness after treatment is completed." c. "You may notice some vaginal relaxation after treatment is completed." d. "You will continue to have normal menstrual periods during treatment."

Ans: B Radiation fields that include the ovaries usually result in premature menopause. Vaginal dryness will occur without estrogen replacement. There should be no sexual intercourse while the implant is in place. Cesium is a radioactive isotope used for therapeutic irradiation of cancerous tissue. There is no documentation to support vaginal relaxation after treatment. Because the client will have premature menopause, she will not have normal menstrual periods

Many older men with prostatic hypertrophy do not seek medical attention until urinary obstruction is almost complete. One reason for this delay in seeking attention is that these men may: a. Feel too self-conscious to seek help when reproductive organs are involved. b. Expect that it is normal to have to live with some urinary problems as they grow older. c. Fear that sexual indiscretions in earlier life may be the cause of their problem. d. Have little discomfort in relation to the amount of pathology because responses to pain stimuli fade with age

Ans: B Research shows that older men tend to believe it is normal to live with some urinary problems. As a result, these men often overlook symptoms and simply attribute them to aging. As part of preventive care for men older than age 40, the yearly physical examination should include palpation of the prostate via rectal examination. Prostate specific antigen screening also is done annually to determine elevations or increasing trends in elevations. The nurse should teach male clients the value of early detection and adequate follow-up for the prostate.

When assessing a client with cervical cancer who had a total abdominal hysterectomy yesterday, you obtain the following data. Which information has the most immediate implications for planning of the client's care? a. Fine crackles are audible at the lung bases. b. The client's right calf is swollen, and she reports calf tenderness. c. The client uses the patient-controlled analgesia device every 30 minutes. d. Urine in the collection bag is amber and clear.

Ans: B Right calf swelling and tenderness indicate the possible presence of deep vein thrombosis. This will change the plan of care, because the client should be placed on bed rest, whereas the usual plan is to ambulate the client as soon as possible after surgery. The other data indicate the need for common postoperative nursing actions such as having the client cough, assessing her pain, and increasing her fluid intake.

The nurse should specifically assess a client with prostatic hypertrophy for which of the following? a. Voiding at less frequent intervals. b. Difficulty starting the flow of urine. c. Painful urination. d. Increased force of the urine stream.

Ans: B Signs and symptoms of prostatic hypertrophy include difficulty starting the flow of urine, urinary frequency and hesitancy, decreased force of the urine stream, interruptions in the urine stream when voiding, and nocturia. The prostate gland surrounds the urethra, and these symptoms are all attributed to obstruction of the urethra resulting from prostatic hypertrophy. Nocturia from incomplete emptying of the bladder is common. Straining and urine retention are usually the symptoms that prompt the client to seek care. Painful urination is generally not a symptom of prostatic hypertrophy

The client with breast cancer is prescribed tamoxifen (Nolvadex) 20 mg daily. The client states she does not like taking medicine and asks the nurse if the tamoxifen is really worth taking. The nurse's best response is which of the following? a. "This drug is part of your chemotherapy program." b. "This drug has been found to decrease metastatic breast cancer." c. "This drug will act as an estrogen in your breast tissue." d. "This drug will prevent hot fl ashes since you cannot take hormone replacement."

Ans: B Tamoxifen is an antiestrogen drug that has been found to be effective against metastatic breast cancer and to improve the survival rate. The drug causes hot flashes as an adverse effect

The client had a mastectomy for cancer of the breast and asks the nurse about a TRAM flap procedure. Which information should the nurse explain to the client? a. The surgeon will insert a saline-filled sac under the skin to simulate a breast. b. The surgeon will pull the client's own tissue under the skin to create a breast. c. The surgeon will use tissue from inside the mouth to make a nipple. d. The surgeon can make the breast any size the client wants the breast to be

Ans: B The TRAM flap procedure is one in which the client's own tissue is used to form the new breast. Abdominal tissue and fat are pulled under the skin with one end left attached to the site of origin to provide circulation until the body builds collateral circulation in the area.

A female with bilateral breast implants asks if she still needs to do breast examinations because she does not know what to feel for. Which of the following is the nurse's best response? a. "Have your partner assess your breasts on a regular basis." b. "I will show you the correct technique as I do the breast examination." c. "A breast examination is very difficult when you have had implant surgery." d. "You need to have a mammogram instead."

Ans: B The client needs to become more confident and knowledgeable about the normal feel of the implants and her breast tissue. The best technique is for the nurse to demonstrate breast self-examination (BSE) to the client as the nurse conducts the clinical breast examination. Implant surgery does not exclude the need for monthly BSE. A mammogram is not a substitute for monthly BSE.

A client with suspected cervical cancer is undergoing a colposcopy with conization. The nurse gives instructions to the client about her menstrual periods, emphasizing that: a. Her periods will return to normal after 6 months. b. Her next two or three periods may be heavier and more prolonged than usual. c. Her next two or three periods will be lighter than normal. d. She may skip her next two periods.

Ans: B The client should be informed that her next two or three periods could be heavy and prolonged. The client is instructed to report any excessive bleeding. The nurse should reinforce the necessity for the follow-up check and the review of the biopsy results with the client. The client's periods will not be normal for 2 to 3 months

The client is diagnosed with breast cancer and is considering whether to have a lumpectomy or a more invasive procedure, a modified radical mastectomy. Which information should the nurse discuss with the client? a. Ask if the client is afraid of having general anesthesia. b. Determine how the client feels about radiation and chemotherapy. c. Tell the client she will need reconstruction with either procedure. d. Find out if the client has any history of breast cancer in her family.

Ans: B The client should understand the treatment regimen for follow-up care. A lumpectomy requires follow-up with radiation therapy to the breast and then systemic chemotherapy. If the cancer is in its early stages, this regimen has results equal to those with a modified radical mastectomy.

The client diagnosed with uterine cancer is complaining of lower back pain and unilateral leg edema. Which statement best explains the scientific rationale for these signs/symptoms? a. This is expected pain for this type of cancer. b. This means the cancer has spread to other areas of the pelvis. c. The pain is a result of the treatment of uterine cancer. d. Radiation treatment always causes some type of pain in the region

Ans: B This pain indicates the cancer is in the retroperitoneal region and the prognosis is poor.

A right orchiectomy is performed on a client with a testicular malignancy. The client expresses concerns regarding his sexuality. The nurse should base the response on the knowledge that the client: a. Is not a candidate for sperm banking. b. Should retain normal sexual drive and function. c. Will be impotent. d. Will have a change in secondary sexual characteristics.

Ans: B Unilateral orchiectomy alone does not result in impotence if the other testis is normal. The other testis should produce enough testosterone to maintain normal sexual drive, functioning, and characteristics. Sperm banking before treatment is commonly recommended because radiation or chemotherapy can affect fertility.

Interdisciplinary management of ovarian cancer includes which of the following? Select all that apply. a. Combination chemotherapy to cure the cancer. b. Bilateral salpingo-oophorectomy to remove diseased organs. c. Radiation therapy to eliminate all cancer cells. d. Referral to social services for supportive care. e. Nutrition therapy for parenteral lipids

Ans: B, D Ovarian cancer is a malignant tumor of the ovary. Ovarian cancer is the fourth most common gynecologic cancer, but the most lethal. It is usually found in advanced stages because it is asymptomatic in early stages. Interdisciplinary management may involve chemotherapy, radiation therapy, surgery and supportive services. Chemotherapy may be used to achieve remission of the disease; it is not, however, curative. Surgery is the treatment of choice, usually involving total hysterectomy with bilateral salpingo-oophorectomy and removal of the omentum. Radiation therapy may be performed for palliative purposes only. The nurse should provide referral to home health services, financial assistance, psychological counseling, clergy, and other social services, as appropriate. Nutrition therapy for parenteral lipids is not part of management of ovarian cancer

When you are developing the plan of care for a home health client who has been discharged after a radical prostatectomy, which activities will you delegate to the home health aide? (Select all that apply.) a. Monitoring the client for symptoms of urinary tract infection b. Helping the client to connect the catheter to the leg bag c. Checking the client's incision for appropriate wound healing d. Assisting the client in ambulating for increasing distances e. Helping the client shower at least every other day

Ans: B, D, E Assisting with catheter care, ambulation, and hygiene are included in home health aide education and would be expected activities for this staff member. Client assessments are the responsibility of N members of the home health care team.

Postoperatively after a modified radical mastectomy, a client has an incisional drainage tube attached to Hemovac suction. The nurse determines the suction is effective when: a. The intrathoracic pressure is decreased, and the client breathes easier. b. There is an increased collateral lymphatic flow toward the operative area. c. Accumulated serum and blood in the operative area are removed. d. No adhesions are formed between the skin and chest wall in the operative area

Ans: C A drainage tube is placed in the wound after a modified radical mastectomy to help remove accumulated blood and fluid in the area. Removal of the drainage fluids assists in wound healing and is intended to decrease the incidence of hematoma, abscess formation, and infection. Drainage tubes placed in a wound do not decrease intrathoracic pressure, increase collateral lymphatic flow, or prevent adhesion formation.

You are assessing a long-term-care client with a history of benign prostatic hyperplasia (BPH). Which information will require the most immediate action? a. The client states that he always has trouble starting his urinary stream. b. The chart shows an elevated level of prostate-specific antigen. c. The bladder is palpable above the symphysis pubis and the client is restless. d. The client says he has not voided since having a glass of juice 4 hours ago.

Ans: C A palpable bladder and restlessness are indicators of urinary retention, which would require action (such as insertion of a catheter) to empty the bladder. The other data would be consistent with the client's diagnosis of BPH. More detailed assessment may be indicated, but no immediate action is required.

You are the charge nurse on the oncology unit. Which client is best to assign to an RN who has floated from the ED? a. Client who needs doxorubicin (Adriamycin) to treat metastatic breast cancer b. Client who needs discharge teaching after surgery for stage Il ovarian cancer c. Client with metastatic prostate cancer who requires frequent assessment and treatment for breakthrough pain d. Client with testicular cancer who requires preoperative teaching about orchiectomy and lymph node resection

Ans: C An RN from the ED would be experienced in assessment and management of pain. Because of their diagnoses and treatments, the other clients should be assigned to RNs who are experienced in caring for clients with cancer.

The 80-year-old male client has been diagnosed with cancer of the prostate. Which treatment should the nurse discuss with the client? a. Radiation therapy every day for four (4) weeks. b. Radical prostatectomy with lymph node dissection. c. Diethylstilbestrol (DES), an estrogen, daily. d. Penile implants to maintain sexual functioning

Ans: C DES is a hormone preparation that suppresses the male hormones and slows the growth of the tumor. Some men with a life expectancy of less than 10 years choose not to treat the cancer at all and will usually die from causes other than prostate cancer

The nurse teaches a female client that the best time in the menstrual cycle to examine the breasts is during the: a. Week that ovulation occurs. b. Week that menstruation occurs. c. First week after menstruation. d. Week before menstruation occurs

Ans: C It is generally recommended that the breasts be examined during the first week after menstruation. During this time, the breasts are least likely to be tender or swollen because estrogen is at its lowest level. Therefore, the examination will be more comfortable for the client. The examination may also be more accurate because the client is more likely to notice an actual change in her breast that is not simply related to hormonal changes

You are working on a medical unit staffed with LPNs/LVNs and UAPs when a client with stage IV ovarian cancer and recurrent ascites is admitted for paracentesis. Which activity is best to delegate to an experienced LPN/LVN? a. Obtaining a paracentesis tray from the central supply area b. Completing the short-stay client admission form c. Measuring vital signs every 15 minutes after the procedure d. Providing discharge instructions after the procedure

Ans: C LPN/LVN education includes vital sign monitoring; an experienced LPN/LVN would recognize and report significant changes in vital signs to the RN. The paracentesis tray could be obtained by a UAP. Client admission assessment and teaching require RN-level education and experience, although part of the data gathering may be done by an LPN/LVN.

A client asks the nurse why the prostate specific antigen (PSA) level is determined before the digital rectal examination. The nurse's best response is which of the following? a. "It is easier for the client." b. "A prostate examination can possibly decrease the PSA." c. "A prostate examination can possibly increase the PSA." d. "If the PSA is normal, the client will not have to undergo the rectal examination."

Ans: C Manipulation of the prostate during the digital rectal examination may falsely increase the PSA levels. The PSA determination and the digital rectal examination are both necessary as screening tools for prostate cancer, and both are recommended for all men older than age 50. Prostate cancer is the most common cancer in men and the second leading killer from cancer among men in the United States. Incidence increases sharply with age, and the disease is predominant in the 60- to 70-year-old age-group

During a client's urinary bladder catheterization, the nurse ensures that the bladder is emptied gradually. The best rationale for the nurse's action is that completely emptying an overdistended bladder at one time tends to cause: a. Renal failure. b. Abdominal cramping. c. Possible shock. d. Atrophy of bladder musculature

Ans: C Rapid emptying of an overdistended bladder may cause hypotension and shock due to the sudden change of pressure within the abdominal viscera. Previously, removing no more than 1,000 mL at one time was the standard of practice, but this is no longer thought to be necessary as long as the overdistended bladder is emptied slowly.

The client has been diagnosed with cancer of the breast. Which referral is most important for the nurse to make? a. The hospital social worker. b. CanSurmount. c. Reach to Recovery. d. I CanCope.

Ans: C Reach to Recovery is a specific referral program for clients diagnosed with breast cancer.

A client with benign prostatic hypertrophy (BPH) has an elevated prostate-specific antigen (PSA) level. The nurse should? a. Instruct the client to have a colonoscopy before coming to conclusions about the PSA results. b. Instruct the client that a urologist will monitor the PSA level biannually when elevated. c. Determine if the prostatic palpation was done before or after the blood sample was drawn. d. Ask the client if he emptied his bladder before the blood sample was obtained

Ans: C Rectal and prostate examinations can increase serum PSA levels; therefore, instruct the client that a manual rectal examination is usually part of the test regimen to determine prostate changes. The prostatic palpation should be done after the blood sample is drawn. The PSA level must be monitored more often than biannually when it is elevated. Having a colonoscopy is not related to the findings of the PSA test. It is not necessary to void prior to having PSA blood levels tested

A 38-year-old female client with a history of breast-conserving surgery, axillary node dissection, and radiation therapy reports that her arm is red, warm to touch, and slightly swollen. Which of the following actions should the nurse suggest? a. Apply warm compresses to the affected arm. b. Elevate the arm on two pillows. c. See the physician immediately. d. Schedule an appointment within 2 to 3 weeks

Ans: C Redness, warmth, and swelling are all signs of infection. Treatment with antibiotics is usually indicated. Infection usually increases fluid accumulation and could worsen the lymphedema. Warm compresses could also increase fluid accumulation. Elevation will not treat the infection. It is critical that the client not delay treatment.

A 68-year-old client who is ready for discharge from the ED has a new prescription for nitroglycerin (Nitrostat) 0.4 mg sublingual as needed for angina. Which client information has the most immediate implications for teaching? a. The client has PH and some urinary hesitancy. b. The client's father and two brothers all have had myocardial infarctions. c. The client uses sildenafil (Viagra) several times weekly for erectile dysfunction. d. The client is unable to remember when he first experienced chest pain.

Ans: C Sildenafil is a potent vasodilator and has caused cardiac arrest in clients who were also taking nitrates such as nitroglycerin. The other client data indicate the need for further assessment and/or teaching, but it is essential for the client who uses nitrates to avoid concurrent use of sildenafil.

The nurse enters the room of a 24-year-old client diagnosed with testicular cancer. The fiancée of the client asks the nurse, "Will we be able to have children?" Which is the nurse's best response? a. "Your fiancée will be able to father children like always." b. "You will have to adopt children because he will be sterile." c. "You and he should consider sperm banking prior to treatment." d. "Have you discussed this with your fiancée? I can't discuss this with you."

Ans: C Sperm banking will allow the client to father children through artificial insemination with the client's sperm.

A client with benign prostatic hypertrophy (BPH) is being treated with terazosin (Hytrin) 2 mg at bedtime. The nurse should monitor the client's: a. Urine nitrites. b. White blood cell count. c. Blood pressure. d. Pulse

Ans: C Terazosin is an antihypertensive drug that is also used in the treatment of BPH. Blood pressure must be monitored to ensure that the client does not develop hypotension, syncope, or orthostatic hypotension. The client should be instructed to change positions slowly. Urine nitrates, white blood cell count, and pulse rate are not affected by terazosin.

Which recommendation is the American Cancer Society's (ACS) 2015 guideline for the early detection of breast cancer? a. Beginning at age 18, have a biannual clinical breast examination by an HCP. b. Beginning at age 30, perform monthly breast self-exams. c. At age 45 through 54, receive a yearly mammogram. d. Beginning at age 50, have a breast sonogram every five (5) years

Ans: C The ACS recommends a yearly mammogram for the early detection of breast cancer beginning at age 45 and going to age 55 and approximately every two (2) years after the age of 55. Before age 45 and after age 55 it should be a discussion between the woman and her HCP to determine if more frequent mammograms are warranted. A mammogram can detect disease that will not be large enough to feel.

A 17-year-old, sexually active female client is seen in the family planning clinic and requests hormonal contraceptives. Before examination, the nurse should explain the importance of regular Papanicolaou (Pap) smears. This recommendation is based on the current screening guidelines of the American Cancer Society for Pap smears, which state that: a. Pap smears are recommended every other year. b. If four consecutive annual Pap smears are negative, the client should schedule repeat Pap smears every 3 years. c. The initial Pap smear should be done at age 21 or earlier if the woman is sexually active. d. If four consecutive smears are negative, the client should request a colposcopy

Ans: C The American Cancer Society guidelines (2004) state that a Pap smear and pelvic examination should be done 3 years after a woman first has vaginal intercourse, but no later than 21 years of age. Annual Pap smears are recommended only for clients at risk and not for the general female population. After three or more consecutive annual examinations with normal findings, the Pap smear may be performed less frequently at the discretion of the physician. Colposcopy is indicated for an abnormal Pap smear, not a negative Pap smear

The client frequently finds lumps in her breasts, especially around her menstrual period. Which information should the nurse teach the client regarding breast self-care? a. This is a benign process, which does not require follow-up. b. The client should eliminate chocolate and caffeine from the diet. c. The client should practice breast self-examination monthly. d. This is the way breast cancer begins and the client needs surgery.

Ans: C The American Cancer Society no longer recommends breast self-examination (BSE) for all women, but it is advisable for women with known breast conditions to perform BSE monthly to detect potential cancer.

The client has had a total abdominal hysterectomy for cancer of the ovary. Which diet should the nurse discuss when providing discharge instructions? a. A low-residue diet without seeds. b. A low-sodium, low-fat diet with skim milk. c. A regular diet with fruits and vegetables. d. A full liquid-only diet with milkshake supplements

Ans: C The client is not placed on a specific diet, but it is always a good recommendation to include fruits and vegetables in the diet.

When preparing a client for discharge 2 days after an abdominal hysterectomy, the nurse should instruct the client to avoid which activities until recovery is complete? a. Swimming in a pool treated with chlorine for 6 weeks after surgery. b. Walking at a leisurely pace for 30 minutes at least once a day. c. Driving until the client can push the brake pedal without pain. d. Lifting greater than 2 lb until the abdominal incision has healed.

Ans: C The client should be prepared for what to expect after surgery. The client should not drive until she can use the brake pedal without abdominal pain. The nurse should teach the client to avoid activities that may increase pelvic congestion, such as dancing or brisk walking, for several months, whereas activities, such as swimming and leisurely walking, may be both physically and mentally helpful. Heavy lifting should be avoided for two months, but the client can lift up to 10 lb as long as there is not tension on the abdomen or abdominal pain.

The client diagnosed with endometriosis experiences pain rated a "5" on a 1-to-10 pain scale during her menses. Which intervention should the nurse teach the client? a. Teach the client to take a stool softener when taking morphine, a narcotic. b. Instruct the client to soak in a tepid bath for 30 to 45 minutes when the pain occurs. c. Explain the need to take the nonsteroidal anti-inflammatory drugs with food. d. Discuss the possibility of a hysterectomy to help relieve the pain

Ans: C The medication of choice for mild to moderate dysmenorrhea is an NSAID. NSAIDs cause gastrointestinal upset and should be taken with food.

The nurse should teach a client that a normal local tissue response to radiation is: a. Atrophy of the skin. b. Scattered pustule formation. c. Redness of the surface tissue. d. Sloughing of two layers of skin

Ans: C The most common reaction of the skin to radiation therapy is redness of the surface tissues. Dryness, tanning, and capillary dilation are also common. Atrophy of the skin, pustules, and sloughing of two layers would not be expected and should be reported to the radiologist.

The nurse writes a client problem of urinary retention for a client diagnosed with Stage IV cancer of the prostate. Which intervention should the nurse implement first? a. Catheterize the client to determine the amount of residual. b. Encourage the client to assume a normal position for urinating. c. Teach the client to use the Valsalva maneuver to empty the bladder. d. Determine the client's normal voiding pattern

Ans: D Determining the client's normal voiding patterns provides a baseline for the nurse and client to use when setting goals.

The nurse is preparing an educational program on breast cancer for women at an African American community center. What information is important for the nurse to consider for the discussion? a. African American women have the lowest rate of breast cancer. b. Most African American women are diagnosed early in the disease process. c. Breast cancer concerns vary between socioeconomic levels of African American women. d. African American women believe breast cancer is inevitable.

Ans: C The nurse needs to consider the beliefs and concerns for all socioeconomic levels of African American women when providing education on breast cancer. Access to screening and care may differ. African American women are more likely to develop breast cancer and be diagnosed later in the disease process than Caucasian women. Not all African American women believe that breast cancer is inevitable.

The client who had a right modified radical mastectomy four (4) years before is being admitted for a cardiac work-up for chest pain. Which intervention is most important for the nurse to implement? a. Determine when the client had chemotherapy last. b. Ask the client if she received Adriamycin, an antineoplastic agent. c. Post a message at the HOB for staff not to use the right arm for venipunctures or BPs. d. Examine the chest wall for cancer sites.

Ans: C The nurse should post a message at the head of the client's bed to not use the right arm for blood pressures or laboratory draws. This client is at risk for lymphedema, and this is a lymphedema precaution.

The client diagnosed with ovarian cancer is prescribed radiation therapy for regional control of the disease. Which statement indicates the client requires further teaching? a. "I will not wash the marks off my abdomen." b. "I will have a treatment every day for six (6) weeks." c. "Nausea caused by radiation therapy cannot be controlled." d. "I need to drink a nutritional shake if I don't feel like eating."

Ans: C There are many medications prescribed for cancer or treatment-induced nausea. The client should notify the HCP if adequate relief is not obtained

When a client is receiving hormone replacement for prostate cancer, the nurse should do which of the following? Select all that apply. a. Inform the client that increased libido is expected with hormone therapy. b. Reassure the client and his significant other that erectile dysfunction will not occur as a consequence of hormone therapy. c. Provide the client the opportunity to communicate concerns and needs. d. Utilize communication strategies that enable the client to gain some feeling of control. e. Suggest that an appointment be made to see a psychiatrist.

Ans: C, D Hormone manipulation deprives tumor cells of androgens or their by-products and, thereby, alleviates symptoms and retards disease progression. Complications of hormonal manipulation include hot flashes, nausea and vomiting, gynecomastia, and sexual dysfunction. As part of supportive care, provide explanations of diagnostic tests and treatment options and help the client gain some feeling of control over his disease and decisions related to it. To help achieve optimal sexual function, give the client the opportunity to communicate his concerns and sexual needs. Inform the client that decreased libido is expected after hormonal manipulation therapy, and that impotence may result from some surgical procedures and radiation. A psychiatrist is not needed.

A client with ovarian cancer asks the nurse, "What is the cause of this cancer?" The most accurate response by the nurse is? a. Use of oral contraceptives increases the risk of ovarian cancer. b. Women who have had at least two live births are protected from ovarian cancer. c. There is less chance of developing ovarian cancer when one lives in an industrialized country. d. The risk of developing ovarian cancer is related to environmental, endocrine, and genetic factors.

Ans: D A definitive cause of carcinoma of the ovary is unknown and the disease is multi-factorial. The risk of developing ovarian cancer is related to environmental, endocrine, and genetic factors. The highest incidence is in industrialized Western countries. Endocrine risk factors for ovarian cancer include women who are nulliparous. Use of oral contraceptives does not increase the risk for developing ovarian cancer but may actually be protective.

A young man with early-stage testicular cancer is scheduled for a unilateral orchiectomy. The client confides to the nurse that he is concerned about what effects the surgery will have on his sexual performance. Which of the following responses by the nurse provides accurate information about sexual performance after an orchiectomy? a. "Most impotence resolves in a couple of months." b. "You could have early ejaculation with this type of surgery." c. "We will refer you to a sex therapist because you will probably notice erectile dysfunction." d. "Because your surgery does not involve other organs or tissues, you'll likely not notice much change in your sexual performance."

Ans: D Although there may not be a big change in sexual function with a unilateral orchiectomy, the loss of a gonad and testosterone may result in decreased libido and sterility. Sperm banking may be an option worth exploring if the number and motility of the sperm are adequate. Remember, the population most affected by testicular cancer is generally young men ages 15 to 34, and in this crucial stage of life, sexual anxieties may be a large concern.

Atropine sulfate is included in the preoperative orders for a client undergoing a modified radical mastectomy. The expected outcome is to: a. Promote general muscular relaxation. b. Decrease pulse and respiratory rates. c. Decrease nausea. d. Inhibit oral and respiratory secretions.

Ans: D Atropine sulfate, a cholinergic blocking agent, is given preoperatively to reduce secretions in the mouth and respiratory tract, which assists in maintaining the integrity of the respiratory system during general anesthesia. Atropine is not used to promote muscle relaxation, decrease nausea and vomiting, or decrease pulse and respiratory rates. It causes the pulse to increase

You are working on the PACU caring for a 32-year-old client who has just arrived after undergoing dilation and curettage to evaluate infertility. Which assessment finding should be immediately communicated to the surgeon? a. Blood pressure of 162/90 mm Hg b. Saturation of the perineal pad after the first 30 minutes c. Oxygen saturation of 91% to 95% d. Sharp, continuous, level 8 (out of 10) abdominal pain

Ans: D Cramping or aching abdominal pain is common after dilation and curettage; however, sharp, continuous pain may indicate uterine perforation, which would require rapid intervention by the surgeon. The other data indicate a need for ongoing assessment or interventions. Transient blood pressure elevation may occur due to the stress response after surgery. Bleeding following the procedure is expected but should decrease over the first 2 hours. And although the oxygen saturation is not at an unsafe level, interventions to improve the saturation should be carried out.

A 56-year-old female is currently receiving radiation therapy to the chest wall for recurrent breast cancer. She has pain while swallowing and burning and tightness in her chest. The nurse should further assess the client for indications of: a. Hiatal hernia. b. Stomatitis. c. Radiation enteritis. d. Esophagitis

Ans: D Difficulty in swallowing, pain, and tightness in the chest are signs of esophagitis, which is a common complication of radiation therapy of the chest wall. Hiatal hernia is a herniation of a portion of the stomach into the esophagus. The client could experience burning and tightness in the chest secondary to a hiatal hernia, but not pain when swallowing. Also, hiatal hernia is not a complication of radiation therapy. Stomatitis is an inflammation of the oral cavity characterized by pain, burning, and ulcerations. The client with stomatitis may experience pain with swallowing, but not burning and tightness in the chest. Radiation enteritis is a disorder of the large and small bowel that occurs during or after radiation therapy to the abdomen, pelvis, or rectum. Nausea, vomiting, abdominal cramping, the frequent urge to have a bowel movement, and watery diarrhea are the signs and symptoms.

A client is beginning external beam radiation therapy to the right axilla after a lumpectomy for breast cancer. Which of the following should the nurse include in client teaching? a. Use a heating pad under the right arm. b. Immobilize the right arm. c. Place ice on the area after each treatment. d. Apply deodorant only under the left arm.

Ans: D Hot, cold, and chemical applications to the area treated should be avoided. The client should be encouraged to use the extremity to prevent muscle atrophy and contractures.

A client is scheduled to undergo transurethral resection of the prostate. The procedure is to be done under spinal anesthesia. Postoperatively, the nurse should assess the client for: a. Seizures. b. Cardiac arrest. c. Renal shutdown. d. Respiratory paralysis.

Ans: D If paralysis of vasomotor nerves in the upper spinal cord occurs when spinal anesthesia is used, the client is likely to develop respiratory paralysis. Artificial ventilation is required until the effects of the anesthesia subside. Seizures, cardiac arrest, and renal shutdown are not likely results of spinal anesthesia.

While performing a breast examination on a 22-year-old client, you obtain the following data. Which finding is of most concern? a. Both breasts have many nodules in the upper outer quadrants. b. The client reports bilateral breast tenderness with palpation. c. The breast on the right side is slightly larger than the left breast. d. An irregularly shaped, nontender lump is palpable in the left breast.

Ans: D Irregularly shaped and nontender lumps are consistent with a diagnosis of breast cancer, so this client needs immediate referral for diagnostic tests such as mammography or ultrasound. The other information is not unusual and does not indicate the need for immediate action.

The day after having a radical prostatectomy, your client has many blood clots in the urinary catheter and states he has frequent bladder spasms. You notice occasional urine leakage around the catheter at the urinary meatus. The client says that his right calf is sore and complains that he feels short of breath. Which action will you take first? a. Irrigate the catheter with 50mL of normal saline b. Administer oxybutynin (Ditropan) 5mg orally c. Apply warm pack to the client's right calf d. Measure oxygen saturation using a pulse oximetry?

Ans: D It is important to assess oxygenation, because the client's calf tenderness and shortness of breath suggest a possible deep vein thrombosis and pulmonary embolus, serious complications of TURP. The other activities are appropriate but are not as high a priority as ensuring that oxygenation is adequate.

Which of the following positions would be best for a client's right arm when she returns to her room after a right modified radical mastectomy with multiple lymph node excisions? a. Across her chest wall. b. At her side at the same level as her body. c. In the position that affords her the greatest comfort without placing pressure on the incision. d. On pillows, with her hand higher than her elbow and her elbow higher than her shoulder.

Ans: D Lymph nodes can be removed from the axillary area when a modified radical mastectomy is done, and each of the nodes is biopsied. To facilitate drainage from the arm on the affected side, the client's arm should be elevated on pillows with her hand higher than her elbow and her elbow higher than her shoulder. A sentinel node biopsy procedure is associated with a decreased risk of lymphedema because fewer nodes are excised.

A client asks the nurse to explain the meaning of her abnormal Papanicolaou (Pap) smear result of atypical squamous cells. Which of the following concepts should the nurse include in the response? a. An atypical Pap smear means that abnormal viral cells were found in the smear. b. An atypical Pap smear means that cancer cells were found in the smear. c. A positive Pap smear alone is not very important diagnostically because there are many false-positive results. d. Abnormal cells in a Pap smear may be caused by various conditions that help identify a problem early.

Ans: D The Pap smear identifies atypical cervical cells that may be present for various reasons. Cancer is the most common possible cause, but not the only one. The Pap smear does not show abnormal viral cells unless specific gene typing is done for human papillomavirus. An adequate smear provides accurate diagnostic data; the false-positive rate is only about 5%

You obtain the following assessment data about your client who has had a transurethral resection of the prostate (TURP) and has continuous bladder irrigation. Which finding indicates the most immediate need for nursing intervention? a. The client states that he feels a continuous urge to void. b. The catheter drainage is light pink with occasional clots. c. The catheter is taped to the client's thigh. d. The client reports painful bladder spasms.

Ans: D The bladder spasms may indicate that blood clots are obstructing the catheter, which would indicate the need for irrigation of the catheter with 30 to 50 mL of normal saline using a piston syringe. The other data would all be normal after a TURP, but the client may need some teaching about the usual post-TURP symptoms and care.

While you are working in the clinic, a healthy 32-year-old woman whose sister is a carrier of the BRCA gene asks you which form of breast cancer screening is the most effective for her. Which response is best? a. "An annual mammogram is usually sufficient screening for women your age. b. "Monthly self-breast examination is recommended because of your higher risk. c. "A yearly breast examination by a health care provider should be scheduled." d. "Magnetic resonance imaging is recommended in addition to annual mammography.

Ans: D The current national guidelines, supported by nonrandomized screening trials and observational data, call for first-degree relatives of clients with the BRCA gene to be screened with both annual mammography and magnetic resonance imaging (MRI). Although annual mammography, breast self-examination, and clinical breast examination by a health care provider may help to detect cancer, the best option for this client is annual mammography and MRI.

The nurse is teaching a class on breast health to a group of ladies at a senior citizen's center. Which risk factor is the most important to emphasize to this group? a. The clients should find out about their family history of breast cancer. b. Men at this age can get breast cancer also and should be screened. c. Monthly breast self-examination is the key to early detection. d. The older a woman gets, the greater the chance of developing breast cancer.

Ans: D The greatest risk factor for developing breast cancer is being female. The second greatest risk factor is being elderly. By age 80, one (1) in every eight (8) women develops breast cancer

A 52-year-old client is scheduled for a total abdominal hysterectomy for cervical cancer. The nurse's discussion regarding the client's feelings and the potential impact of this procedure on her sexuality should include which of the following questions? a. "All women experience sexual problems with this surgical procedure. Do you have any questions?" b. "When can I schedule an appointment with you and your partner to discuss any issues either of you may have regarding sexuality?" c. "Do you anticipate any problems with sex related to your scheduled hysterectomy?" d. "Most women have concerns about their sexuality after this type of surgery. Do you have any concerns or questions?"

Ans: D This question introduces some basic information and allows for support for the client who may be experiencing some sexuality concerns. Not all women experience sexual problems after undergoing a hysterectomy. Assuming that the client will want to schedule an appointment with her partner is inappropriate and may embarrass her. Simply asking the client whether she expects to have problems with sex is too abrupt and does not provide any information.

The client is eight (8) hours post-transurethral prostatectomy for cancer of the prostate. Which nursing intervention is priority at this time? a. Control postoperative pain. b. Assess abdominal dressing. c. Encourage early ambulation to prevent DVT. d. Monitor fluid and electrolyte balance

Ans: D With irrigation of the surgical site through the indwelling three (3)-way catheter to prevent blood clots, fluids may be absorbed through the open surgical site and retained. This can lead to fluid volume overload and electrolyte imbalance (hyponatremia).

Which of the following is a risk factor for cervical cancer? a. Sexual experience with one partner. b. Sedentary lifestyle. c. Obesity. d. Adolescent pregnancy

Ans: D Young age at first pregnancy is a risk factor for cervical cancer. Other risk factors include a family history of the disease, sexual experience with multiple partners, and a history of sexually transmitted disease (e.g., syphilis, human papillomavirus infection, gonorrhea). Cigarette smoking, promiscuous male partner, human immunodeficiency virus infection or other immunosuppression, and low socioeconomic status are other risk factors. Sexual relations with one partner, sedentary lifestyle, and obesity are not risk factors for cervical cancer.


संबंधित स्टडी सेट्स

2.1-2.10 AP-Style MC Practices & Quizzes

View Set

Human Sexuality: Making Informed Decisions Exam 1

View Set

MTA 98-361 Ch.5 Understanding Desktop Applications Questions

View Set

DH244 interpretation of dental caries

View Set

Listing and Selling HUD Homes (h)

View Set